46-я Международная Математическая Oлимпиада
Мексика, Мериде, 2005 год


Пусть $x,y,z$ — такие положительные числа, что $xyz > 1$. Докажите, что $\dfrac{{{x}^{5}}-{{x}^{2}}}{{{x}^{5}}+{{y}^{2}}+{{z}^{2}}}+\dfrac{{{y}^{5}}-{{y}^{2}}}{{{x}^{2}}+{{y}^{5}}+{{z}^{2}}}+\dfrac{{{z}^{5}}-{{z}^{2}}}{{{x}^{2}}+{{y}^{2}}+{{z}^{5}}}\ge 0.$
посмотреть в олимпиаде

Комментарий/решение:

пред. Правка 4   2
2021-03-30 20:20:55.0 #

Теңсіздікті келесідей түрлендіреміз:

$$\frac{x^5-x^2}{x^5+y^2+z^2}- \frac{x^5-x^2}{x^3(x^2+y^2+z^2)}=\frac{(x^5-x^2)\cdot x^3(x^2+y^2+z^2)-(x^5-x^2)(x^5+y^2+z^2)}{x^3(x^2+y^2+z^2)(x^5+y^2+z^2)}=\frac{x^2(x^3-1)(x^5+x^3y^2+x^3y^2-x^5-y^2-z^2)}{x^3(x^2+y^2+z^2)(x^5+y^2+z^2)}=\frac{x^2(x^3-1)(y^2(x^3-1)+z^2(x^3-1))}{x^3(x^2+y^2+z^2)(x^5+y^2+z^2)}=\frac{x^2(x^3-1)^2(y^2+z^2)}{x^3(x^2+y^2+z^2)(x^5+y^2+z^2)}\geq 0.$$

Cондықтан:

$$\frac{x^5-x^2}{x^5+y^2+z^2}\geq \frac{x^5-x^2}{x^3(x^2+y^2+z^2)}\geq \frac{x^5-x^2\cdot xyz}{x^3(x^2+y^2+z^2)}= \frac{x^2-yz}{(x^2+y^2+z^2)}.$$

Қалған екеуі үшін де осылай түрлендіру жасаймыз: $\frac{y^5-y^2}{x^2+y^5+z^2}\geq \frac{y^2-zx}{x^2+y^2+z^2}$ және $\frac{z^5-z^2}{x^2+y^2+z^5}\geq \frac{z^2-xy}{x^2+y^2+z^2}.$

Пайда болған теңсіздіктерді қоссақ: $$\frac{x^5-x^2}{x^5+y^2+z^2}+\frac{y^5-x^2}{x^2+y^5+z^2}+\frac{z^5-z^2}{x^2+y^2+z^5}\geq \frac{x^2+y^2+z^2-xy-yz-zx}{x^2+y^2+z^2}\geq 0.$$

$x^2+y^2+z^2-xy-yz-zx \geq 0$ теңсіздігін дәлелдесек: екі жағын $2$-ге көбейтеміз:

$(x^2+y^2)+(y^2+z^2)+(x^2+x^2)-2xy-2yz-2xz \geq 0.$

$(x-y)^2+(y-z)^2+(x-z)^2 \geq 0.$

  3
2021-03-30 23:19:47.0 #

Решение: Заметим, что условие равносильно с неравенством

$$\sum\dfrac{x^2+y^2+z^2}{x^5+y^2+z^2}\le 3,$$

с другой стороны из КБШ: $\left(x^5+y^2+z^2\right)\left(\dfrac{1}{x}+y^2+z^2\right)\ge \left(x^2+y^2+z^2\right)^2\iff \dfrac{x^2+y^2+z^2}{x^5+y^2+z^2}\le \dfrac{\dfrac 1 x +y^2+z^2}{x^2+y^2+z^2},$ откуда

$$\sum\dfrac{x^2+y^2+z^2}{x^5+y^2+z^2}\le \sum\dfrac{\dfrac 1 x +y^2+z^2}{x^2+y^2+z^2}=\dfrac{\dfrac 1 x +\dfrac 1 y+\dfrac 1 z}{x^2+y^2+z^2}+2=\dfrac{xy+yz+zx}{xyz\left(x^2+y^2+z^2\right)}+2\le \dfrac{x^2+y^2+z^2}{xyz(x^2+y^2+z^2)}+2\le 1+2=3.\ \square$$

  1
2021-04-01 20:01:29.0 #

Отличное решение!